The Fundamental Theorem for Line Integrals

Click For Summary
The discussion focuses on determining if the vector field f(x,y) = <-3e^(-3x)sin(-3y), -3e^(-3x)cos(-3y)> is conservative. It confirms that f is a conservative vector field and discusses the need to find a potential function f such that F = ∇f. The process involves integrating the components of the vector field to derive the potential function. The integration of one component leads to a function with a constant of integration that varies with respect to the other variable. The key takeaway is that solving the equations for f(x,y) requires careful integration and consideration of constants.
Chas3down
Messages
60
Reaction score
0

Homework Statement


Determine whether or not f(x,y) is a conservative vector field.
f(x,y) = <-3e^(-3x)sin(-3y),-3e^(-3x)cos(-3y) >

If F is a conservative fector field find F = gradient of f

Homework Equations


N/A

The Attempt at a Solution



Fx = -3e^(-3x)(-3)cos(-3y)
Fy = -3e^(-3x)(-3)cos(-3y)

f is a conservative fector field
(This part is all correct.)

F = ? + K

It won't accept a vector, I know how to normally find a gradient vector, but that returns a vector, I need a non-vector answer..
 
Last edited:
Physics news on Phys.org
<br /> \vec \nabla f=&lt;\partial_x f,\partial_y f&gt;=\vec F=&lt;F_x,F_y&gt; \Rightarrow \left\{ \begin{array}{c} f=\int F_x dx \\ f=\int F_y dy \end{array} \right.<br />
 
Since ##\vec F(x,y)## is conservative, you know ##\vec F(x,y) = \vec{\nabla f(x,y)}## for some potential function ##f##.

This amounts to saying:

##\vec F(x,y) = \vec{\nabla f(x,y)}##
##P \hat i + Q \hat j = f_x \hat i + f_y \hat j##

Equating the vector components: ##P = f_x## and ##Q = f_y##.

So really you want to solve those two equations for some ##f(x,y)##. The usual method would be to integrate one of those two equations to find some ##f(x,y)## that has a constant of integration which varies. For example, taking the first:

$$f(x,y) = \int P dx = P' + g(y)$$

Afterwards, you should look at the equation you didn't use and observe the outcome.
 
Question: A clock's minute hand has length 4 and its hour hand has length 3. What is the distance between the tips at the moment when it is increasing most rapidly?(Putnam Exam Question) Answer: Making assumption that both the hands moves at constant angular velocities, the answer is ## \sqrt{7} .## But don't you think this assumption is somewhat doubtful and wrong?

Similar threads

  • · Replies 8 ·
Replies
8
Views
2K
  • · Replies 10 ·
Replies
10
Views
2K
Replies
3
Views
2K
  • · Replies 3 ·
Replies
3
Views
2K
  • · Replies 8 ·
Replies
8
Views
2K
  • · Replies 2 ·
Replies
2
Views
2K
Replies
7
Views
2K
  • · Replies 4 ·
Replies
4
Views
2K
  • · Replies 1 ·
Replies
1
Views
4K
  • · Replies 8 ·
Replies
8
Views
2K